Đến nội dung

tuan101293 nội dung

Có 316 mục bởi tuan101293 (Tìm giới hạn từ 30-05-2020)



Sắp theo                Sắp xếp  

#307939 $$f(f(m)+f(n))=m+n,\forall m,n \in \mathbb{N^*}...

Đã gửi bởi tuan101293 on 03-04-2012 - 15:04 trong Phương trình hàm

+,Dễ thấy f đơn ánh và với mọi $n\ge 2$ thì tồn tại s để $f(s)=n$
+,thay (m+1,n-1) vào ta có :$f(m+1)-f(m)=f(n)-f(n-1)$ nên suy ra $f(m+1)-f(m)=f(2)-f(1)$
dễ thấy f(2)>f(1) nếu ko f(n)<f(1) (vô lý)
suy ra f(m+1)>f(m). xét tiếp số s mà f(s)=2. ta thấy nếu $s\ge 3$ suy ra f(2)<2 hay f(1)<1 loại nên ta có 2 TH
+,f(2)=2 suy ra f(n)=n
+,f(1)=2 suy ra thay (1,1) vào đề bài ta có f(2f(1))=2=f(1) hay f(1)=1/2, loại
suy ra f(n)=n



#455477 $\sqrt[3]{x-9}= (x-3)^3+6$

Đã gửi bởi tuan101293 on 05-10-2013 - 22:29 trong Phương trình - hệ phương trình - bất phương trình

viet lại pt $\sqrt[3]{x-9}+(x-9) = (x-3) + (x-3)^3$

nếu ta đặt $f(x) = x^3 + x$ (f là hàm tăng) ta có $f(\sqrt[3]{x-9}) = f(x-3)$

hay $\sqrt[3]{x-9} = x-3$, mũ 3 lên phân tích ta có x=1 và $x= 4+- i\sqrt{2}$




#456652 $\sum(\frac{ab}{3b+c+d+3} \leq frac...

Đã gửi bởi tuan101293 on 10-10-2013 - 20:27 trong Bất đẳng thức - Cực trị

Bài này ghép ghép vui phết

Nhân 2 vế với 9  và dùng bdt svac 

ta có $$\frac{9ab}{a+4b+2c+2d} \le \frac{ab}{a+2c} + \frac{2ab}{2b+d}$$

         $$\frac{9bc}{b+4c+2d+2a} \le \frac{2bc}{a+2c} +\frac{bc}{b+2d}$$

         $$\frac{9cd}{c+4d+2a+2b} \le \frac{2cd}{b+2d} +\frac{cd}{c+2a}$$

         $$\frac{9da}{d+4a+2b+2c} \le \frac{2da}{2a+c} +\frac{da}{d+2b}$$

Cộng dọc ta có $$9LHS \le a+b+c+d = 3$$

q,e,d




#454596 $2n^{2}-1$

Đã gửi bởi tuan101293 on 02-10-2013 - 01:31 trong Số học

Bài toán vẫn đúng nếu với n bất kỳ, là 1 hệ quả của định lý Cauchy Davenport.

có thể tham khảo lời giải tại đây

http://www.tau.ac.il...a/PDFS/egz1.pdf




#454392 $f(f(x))=x \wedge f(f(x)+1)= 1- x $

Đã gửi bởi tuan101293 on 01-10-2013 - 04:37 trong Phương trình hàm

Câu trả lời là không tồn tại nhé:

f(f(x)) = x suy ra f là song ánh.

$x=1-(1-x)=f(f(1-x)+1)$ suy ra $f(x)=f(1-x)+1$, thay 1-x vào cái này ta có $f(1-x)=f(x)+1$ vô lý




#454393 $u_{1}=1;u_{2}=2;u_{n+1}=\dfrac{...

Đã gửi bởi tuan101293 on 01-10-2013 - 05:31 trong Dãy số - Giới hạn

Theo em thì có lẽ bài này sai đề, với lại nếu đúng đi nữa thì số hạng thứ 5 cũng không thuộc N nên có lẽ không cần quan tâm cho mệt @@




#454583 $x^{4}-y^{4}=240$

Đã gửi bởi tuan101293 on 01-10-2013 - 23:11 trong Phương trình - Hệ phương trình - Bất phương trình

Bài này là bài 1 VMO 2010 

http://www.artofprob...533ba0#p2103782

ý tưởng là lấy pt trên trừ 8 pt dưới đề quy về bậc 4




#454170 $x_1=a (a> -1); x_{n+1}=\frac{1}{1+x_n...

Đã gửi bởi tuan101293 on 30-09-2013 - 00:16 trong Dãy số - Giới hạn

đặt $$x_n = \frac{y_n}{z_n}$$ với $$y_1 = a, z_1 = 1$$

ta có công thức $$\frac{y_{n+1}}{z_{n+1}} = \frac{z_{n}}{y_{n}+z_{n}}$$

nên công thức truy hồi cho 2 dãy là $$y_{n+1} = z_{n} , z_{n+1} = y_{n}+z_{n}$$ hay $$y_{n+1} = z_{n}, z_{n+1} = z_{n} + z_{n-1}$$

nên $$z_{n}$$ là dãy Fibonacci và $$x_{n}$$ chính là tỷ lệ giữa 2 số Fibo liên tiếp nên dãy hội tụ và $$lim = \frac{\sqrt{5}-1}{2}$$

bài 2 thì tương tự 




#220903 1 bai toan luong giac

Đã gửi bởi tuan101293 on 19-11-2009 - 22:38 trong Các dạng toán khác

chung minh:
cos72= :frac{3- :sqrt{5} }{2( :sqrt{5} -1} chua biet cach go cong thuc mong moi nguoi thong cam

Bài này có trong sbt lớp 10 mà
CM bằng cách dựng tam giác có 3 góc là 72,72,36....



#244799 1 bài toán khó

Đã gửi bởi tuan101293 on 22-10-2010 - 19:35 trong Các bài toán Đại số khác

Em đọc kỹ xem,đúng đó,ko cần chia nhiều TH đâu:D



#244740 1 bài toán khó

Đã gửi bởi tuan101293 on 22-10-2010 - 14:19 trong Các bài toán Đại số khác

Cho f là hàm số lẻ. Hàm số ĐB trên R.
Nếu a+b+c=0. Chứng minh rằng

f(a).f(b)+f(b).f(c )+f(c ).f(a) :Rightarrow 0
Chứng minh giùm em với!

Em đang cần rất gấp

+, f là hàm lẻ suy ra $f(x)=-f(-x)$ nên f(0)=0
+, Giả sử $a\ge b\ge c$
suy ra ta phải CM
$f(a)f(b)\le (f(b)+f(a))f(a+b)$
Ta có $f(a)+f(b)\ge 0$
và $f(a+b)\ge f(b)$ do $a\ge 0$
suy ra $(f(a)+f(b))f(a+b)\ge (f(a)+f(b))f(b)\ge f(a)f(b)$
ĐPCM



#257965 1 bài về đa thức và BĐT

Đã gửi bởi tuan101293 on 13-04-2011 - 20:37 trong Bất đẳng thức - Cực trị

Cho đa thức $P(x)=x^{n}+a_1x^{n-1}+a_2x^{n-2}+....+a_n(n \ge 2)$ có nghiệm thực $b_1,b_2,...,b_n$.Chứng minh rằng:
$P(x+1) \sum\limits_{i=1}^{n}\dfrac{1}{x-b_{i}} \ge 2n^2,\forall x>b_{i}(i=\overline{1,n})$

Có $P(x)=\prod (x-b_i)$
nên ta đặt $x-b_i=a_i$, suy ra $a_i>0$ quy về việc CM
$(1+a_1)(1+a_2)....(1+a_n)(\dfrac{1}{a_1}+....+\dfrac{1}{a_n})\ge 2n^2$
holder+cosi 1 fat,quy về việc CM
$(1+x)^n\ge 2nx$ với $x=\sqrt[n]{a_1a_2....a_n}$
(hiển nhiên đúng)
đpcm



#241575 1 Bất đẳng thức số học

Đã gửi bởi tuan101293 on 21-09-2010 - 18:00 trong Bất đẳng thức và cực trị

Đặt a,b,c cho nhanh nhé ta có $\dfrac{b+c}{a},\dfrac{c+a}{b},\dfrac{a+b}{c}\in N*$
Giả sử a=max(a,b,c) thì suy ra $b+c\le 2a$ suy ra $b+c=2a$ hoắc $b+c=a$
TH1:b+c=2a suy ra a=b=c thì tổng kia =6<8 đúng
TH2:b+c=a, suy ra c=a-b thay nốt vào 2 cái kia ta được
+,$b|a+a-b$ suy ra $b|2a$ (1)
+,$(a-b)|a+b$ suy ra $a-b|2b$ suy ra $a\le 3b$ (2)
chú ý $a=b+c\ge b+1$ nên từ 1 và 2 suy ra
$a\in {\dfrac{3b}{2},2b,\dfrac{5b}{2},3b}$ (chú ý cái 3b/2 và 5b/2 là với b chẵn)
thay lại vào cái $a-b|2b$ thấy cái 5b/2 bị kích vì ko thỏa mãn.
còn lại ta thay nốt các bộ sau vào đề tính tổng: $(a,b,c)=(\dfrac{3b}{2},b,\dfrac{b}{2}),(2b,b,b),(3b,b,2b)$ thấy tổng =8,7,8
nên ta có ĐPCM



#454637 1 số bài toán vui

Đã gửi bởi tuan101293 on 02-10-2013 - 14:31 trong IQ và Toán thông minh

1, 3 người mà 2 người bất kỳ đều bị nhọ  thì cả 3 đều nhọ.

2, Nếu cô thứ 1 nói đúng suy ra cô 2 là Nhị và nói sai vào thứ 2, vô lý

    suy ra cô thứ 1 nói sai, cô thứ 1 là Nhị và ngày hôm đó là 3,5,7

   cô thứ 2 là Nhất, thứ 4 nói dối tức là cô ta cũng nói dối nên ngày hôm đó là thứ 3.

3, Cụ nói là: chỉ cần nhảy lên ngựa người kia và phi về đích là thắng mà. 




#220443 1/a^2+2bc

Đã gửi bởi tuan101293 on 15-11-2009 - 18:15 trong Bất đẳng thức - Cực trị

Với cùng điệu kiện trên thì bất đẳng thức sau vẫn đúng :

$\dfrac{1}{{{a^2} + 2bc}} + \dfrac{1}{{{b^2} + 2ca}} + \dfrac{1}{{{c^2} + 2ab}} \ge \dfrac{{ab + bc + ca}}{{{a^2}{b^2} + {b^2}{c^2} + {c^2}{a^2}}} + \dfrac{{37}}{2}.\dfrac{{{{\left[ {\left( {a - b} \right)\left( {b - c} \right)\left( {c - a} \right)} \right]}^2}}}{{{{\left( {a + b + c} \right)}^6}\left( {ab + bc + ca} \right)}}$

Mình nghĩ là bdt này có vấn đề
chắc là phải thay 37 thành 27 và 1 số cái nữa



#220435 1/a^2+2bc

Đã gửi bởi tuan101293 on 15-11-2009 - 17:25 trong Bất đẳng thức - Cực trị

mình giải qua thế này nhé
Ta xét TH a,b,c>0
chú ý là
$\sum_{cyc}\dfrac{1}{a^2+2bc}=\dfrac{(\sum ab)(2\sum a^2+\sum ab)}{\prod(a^2+2bc)}$
tức là ta sẽ CM
$(2\sum a^2+\sum ab)(\sum a^2b^2)\ge \prod(a^2+2bc)$
(mình định thử côsi nhưng mà phải ăn cơm nên làm vội phân tích kiểu SOS và schur)
expand ra ta phải CM
$\sum_{cyc}2a^4(b-c)^2-\sum a^2b^2(c-b)(c-a)\ge 0$
đúng theo côsi 2 số
ĐPCM



#220917 1/a^2+2bc

Đã gửi bởi tuan101293 on 20-11-2009 - 08:32 trong Bất đẳng thức - Cực trị

Tớ gõ các này mục đích là tập tành cái chuyện soạn văn bản bằng latex thôi :(

Tớ lần đầu gõ latex nên mò mẫm có hơi lâu :leq ( độ .... 4 -5 tiếng gì đấy :leq ) , lệnh thì tớ biết rất ít ---> trình bày có hơi khó coi 1 xíu mong mọi người bỏ qua cho :luoi .

Cái lời giải này của VIMF tớ cũng chưa check nữa ... anh em xem có thấy cái gì sai thì nói tớ sửa nhá :luoi

Thì mình cũng bảo là 27 thì có lời giải đẹp mà:leq
còn cái 310 thì nhìn kinh quá,ko dám expand.
thks
p/s:lời giải quy đồng của mình đẹp hơn:D



#220840 1/a^2+2bc

Đã gửi bởi tuan101293 on 19-11-2009 - 16:25 trong Bất đẳng thức - Cực trị

Thay đổi một chút nhá!!!! Hãy cm:
$\dfrac{1}{2a^2+bc}+\dfrac{1}{2b^2+ca}+\dfrac{1}{2c^2+ab} \ge \dfrac{ab+bc+ca}{a^2b^2+b^2c^2+c^2a^2}$
:(

Công nhận là thế thật:
ta có $a^2b^2+b^2c^2+c^2a^2\ge 2bca^2+b^2c^2=bc(2a^2+bc)$
tương tự ghép vào ta có đpcm



#252757 22222

Đã gửi bởi tuan101293 on 03-02-2011 - 20:09 trong Bất đẳng thức - Cực trị

Cho $X,Y,Z , A,B,C$ là các số thực không âm thỏa mãn:

$A+X=B+Y=C+Z=k$

cmr:

$AY+BZ+CX\le k^2$

Mở rộng bài toán!

cho k=1,ta phải CM
với $a,b,c\in [0,1]$ thì
$a+b+c-ab-bc-ca\le 1$
mà $(1-a)(1-b)(1-c)\ge 0$ nên $1-a-b-c+ab+bc+ca\ge abc\ge 0$
ĐPCM



#239367 567 Nice And Hard Inequalities

Đã gửi bởi tuan101293 on 03-09-2010 - 20:47 trong Tài nguyên Olympic toán

Một tài liệu Bất đẳng thức Olympic của mình đây.
Tặng bạn Curi gem.
567 Nice And Hard Inequalities

Vì một số lí do mà Messi_ndt chưa hoàn chỉnh được tài liệu này được, Up lên cho anh em tham khảo.
Vẫn còn một số chỗ chưa hoàn chỉnh và thiếu một số phần mà mình dư định làm.
Dù sao đây cũng là tâm huyết của Messi_ndt trong một thời giãn dài.
Anh em dowdload về đọc thử nhé.
Các bạn có thể dowload ở đây.


Xem qua file này mà anh thấy mình gà bdt quá.:geq
thks em nhé.



#241791 [Giải Phương Trinh] Dạng tìm M để phương trình có nghiệm

Đã gửi bởi tuan101293 on 24-09-2010 - 14:15 trong Các bài toán Đại số khác

Mai kiểm tra mà ông thầy ra đề hay quá. Em làm ko nổi. Mấy pác giải hộ em bài này với.

1. Cho phương trinh x^4 + x^3 - 2x^2 + 3mx - m^2 = 0

Tìm m để phương trinh có nghiệm
Thank trc mấy pác. Giúp em nghen

Dạng này thì ngồi phân tích thôi
ta có $ x^4 + x^3 - 2x^2 + 3mx - m^2 =(x^2-x+m)(x^2+2x-m)$
đến đây thì ok rồi



#238905 a,b,c

Đã gửi bởi tuan101293 on 31-08-2010 - 22:54 trong Bất đẳng thức và cực trị

Bài này em hỏi anh luat nhé.
hướng là dùng bdt này :
$\sum ab\ge \sum \dfrac{4a^3(b+c-1)}{(b+c)^2}$



#226080 Đa thức

Đã gửi bởi tuan101293 on 14-01-2010 - 12:56 trong Phương trình - Hệ phương trình - Bất phương trình

Mình nói ý nhé:
bài này là định lý fejer(mình cũng ko bik tên tiêg anh nữa:D)
CM bằng cách đạo hàm, chú ý là có công thức tính tổng \sigma cos(k) là ra thôi



#226042 Đa thức

Đã gửi bởi tuan101293 on 13-01-2010 - 20:01 trong Phương trình - Hệ phương trình - Bất phương trình

bai 1 du 100
bai 2 thi chac la chia cho x^2,roi dung bunhia.mih chua tinh ky lam.:geq.



#221012 Đa thức

Đã gửi bởi tuan101293 on 20-11-2009 - 19:43 trong Phương trình - Hệ phương trình - Bất phương trình

Cho đa thức f(x) bậc n có n nghiệm thực .a là một nghiệm bội của f'(x).CMR:f(a)=0